You are on page 1of 7

Solutions for Math 311 Assignment #1

(1) Show that


(a) Re(iz) = Im(z);
(b) Im(iz) = Re(z).
Proof. Let z = x + yi with x = Re(z) and y = Im(z). Then
Re(iz) = Re(y + xi) = y = Im(z) and Im(iz) = Im(y +
xi) = x = Re(z).

(2) Verify the associative law for multiplication of complex numbers. That is, show that
(z1 z2 )z3 = z1 (z2 z3 )
for all z1 , z2 , z3 C.
Proof. Let zk = xk + iyk for k = 1, 2, 3. Then
(z1 z2 )z3 = ((x1 + y1 i)(x2 + y2 i))(x3 + y3 i)
= ((x1 x2 y1 y2 ) + i(x2 y1 + x1 y2 ))(x3 + y3 i)
= (x1 x2 x3 x3 y1 y2 x2 y1 y3 x1 y2 y3 )
+ i(x2 x3 y1 + x1 x3 y2 + x1 x2 y3 y1 y2 y3 )
and
z1 (z2 z3 ) = (x1 + y1 i)((x2 + y2 i))(x3 + y3 i))
= (x1 + y1 i)((x2 x3 y2 y3 ) + i(x2 y3 + x3 y2 ))
= (x1 x2 x3 x3 y1 y2 x2 y1 y3 x1 y2 y3 )
+ i(x2 x3 y1 + x1 x3 y2 + x1 x2 y3 y1 y2 y3 )
Therefore,
(z1 z2 )z3 = z1 (z2 z3 )

(3) Compute
1+i
(a)
;
1i
(b) (1 i)4 .
Answer. (a) i (b) 4
(4) In each case, sketch the set of points on the complex plane
determined by:
(a) |z i| = 2;
1

(b) 2 < |z + 3 + i| < 3;


(c) |z 1| = |z 2i|;
(d) z 2 + z 2 = 2;
(e) | Im(z + i)| > 3.
Answer. (a) The circle x2 + (y 1)2 = 4.
(b) The annulus 4 (x + 3)2 + (y + 1)2 < 9.
(c) The line 2x + 4y + 3 = 0.
(d) The hyperbola x2 y 2 = 1.
(e) The union of two half planes {y > 4} and {y < 2}.
(5) Show that
Im(z1 z2 )
|z1 | + |z2 |

|z3 + z4 |
||z3 | |z4 ||
for all complex numbers z1 , z2 , z3 , z4 satisfying |z3 | 6= |z4 |. Also
give the sufficient and necessary condition under which the
equality holds.
Proof. Since
| Im(z1 z2 )| |z1 z2 | |z1 + z2 |
and


|z3 + z4 | |z3 | |z4 |
we have
Im(z1 z2 )
| Im(z1 z2 )|
|z1 | + |z2 |
|z1 | + |z2 |

.
|z3 + z4 |
|z3 + z4 |
||z3 | |z4 ||
||z3 | |z4 ||
If the equality holds, we must have Re(z1 z2 ) = 0, Im(z1 z2 )
0 and |z1 z2 | = |z1 | + |z2 |. It follows that x1 = x2 = 0, y1 y2
and y1 y2 0, where we let zk = xk + iyk for k = 1, 2, 3, 4.
If z1 6= 0 or z2 6= 0, we also have |z3 + z4 | = |z3 | |z4 | , i.e.,
z3 = z4 or z4 = z3 for some < 0.
In conclusion, the inequality holds if x1 = x2 = 0, y1 y2 ,
y1 y2 0 and
z1 = z2 = 0 or
z3 = z4 or
z4 = z3 for some < 0.

(6) Use
(a)
(b)
(c)

the properties of conjugates and moduli to show that


z + 3i = z 3i;
iz = iz;

|(2z + 5)( 2 i)| = 3|2z + 5|.

Proof. (a)
z + 3i = z + 3i = z 3i.
(b)
iz = iz = iz
(c)

|(2z + 5)( 2 i)| = |2z + 5|| 2 i|

= 3|2z + 5| = 3|2z + 5|


Solutions for Math 311 Assignment #2


(1) Find the principal argument Arg(z) when
i
(a) z =
;
1+ i
(b) z = ( 3 + i)2010 .
Solution. (a)




i
1 i
Arg
+
= Arg
= Arg
1+i
2 2

2 i/4
e
= /4
2

(b)

Arg(( 3 + i)2010 ) = Arg 22010

3 i
+
2
2

!2010

= Arg((ei/6 )2010 ) = Arg(e335i ) = Arg(ei ) =


(2) Show that if Re(z1 ) > 0 and Re(z2 ) > 0, then
Arg(z1 z2 ) = Arg(z1 ) + Arg(z2 ).
Proof. We always have
Arg(z1 z2 ) = Arg(z1 ) + Arg(z2 ) + 2k
for some integer k. Since Re(z1 ) > 0 and Re(z2 ) > 0,
/2 < Arg(z1 ) < /2 and /2 < Arg(z2 ) < /2
and hence
< Arg(z1 ) + Arg(z2 ) < .
And since
< Arg(z1 z2 ) ,

2 < Arg(z1 z2 ) (Arg(z1 ) + Arg(z2 )) < 2


Therefore, 2 < 2k < 2 and 1 < k < 1. Hence k = 0. 
(3) Use the identity
n
X
k=0

to derive
n
X

sin(k) =

k=1

zk =

1 z n+1
1z

cos(/2) cos((2n + 1)/2)


2 sin(/2)

Proof. Let z = ei . Then


n
n
n
X
X
X
1 ei(n+1)
ik
cos(k) + i
sin(k) =
e =
1 ei
k=0
k=1
k=0
1 ei(n+1)
1 ei(n+1)
=
(1 cos ) i sin
2 sin(/2)(sin(/2) i cos(/2))
1 ei(n+1)
i(ei/2 e(2n+1)i/2 )
=
=
2 sin(/2)(i)ei/2
2 sin(/2)
(sin((2n + 1)/2) + sin(/2)) + i(cos(/2) cos((2n + 1)/2)
=
2 sin(/2)
=

Therefore,
n
X
k=1

sin(k) =

cos(/2) cos((2n + 1)/2


.
2 sin(/2)


(4) Find all the square roots of (a) 1 + i and (b) 1 +


them in rectangular coordinates.

3i. Express

Solution. (a)
q

4
4
1+i=
2ei/4 = 2eki+i/8 = 2ei/8
p

By
half-angle
formula,
cos(/8)
=
4
+
2/2 and sin(/8) =
p

4 2/2. Therefore,

q

q
4

2
1+i=
4+ 2+i 4 2
2

(b)
q

1 + 3i = 2 ei/3 = 2eki+i/6 = 2ei/6 =

!
6
2
+i
2
2

(5) Find the four zeros of the polynomial z 4 + 4 and use these to
factor z 4 + 4 into quadratic factors with real coefficients.
Solution. The 4th roots of 4 are



ki i
4
4
i
4 = 2 e = 2 exp
+
2
4
for k = 0, 1, 2, 3. So

z 4 + 4 = (z 2ei/4 )(z 2e3i/4 )(z 2e5i/4 )(z 2e7i/4 )

= (z 2ei/4 )(z 2ei/4 )(z 2e3i/4 )(z 2e3i/4 )

= (z 2 2(ei/4 + ei/4 )z + 2)(z 2 2(e3i/4 + e3i/4 )z + 2)


= (z 2 2z + 2)(z 2 + 2z + 2)
(6) Sketch the following sets and determine which are domains,
which are closed and which are bounded:
(a) |z 2 + i| 1;
(b) |2z + 3| > 4;
(c) Im(z) > 1;
(d) |z 4| |z|;
(e) Im(z) = 1;
(f) 0 arg(z) /4(z 6= 0).
Justify your answer.
Solution. (a) It is not a domain since it is not open; choose a
point z0 satisfying |z0 2+i| = 1 and the disk |z z0 | < r is not
contained in the set for all r > 0. It is closed since {|z 2 + i| >
1} is open and it is bounded since |z| |2 i| + 1 < 4 for all z
satisfying |z 2 + i| 1.
(b) It is a domain since it is open and connected. It is not
closed since {|2z + 3| 4} is not open and it is not bounded
since one can find a sequence zn satisfying |2zn + 3| > 4 such
that limn |zn | = . For example, let zn = n.
(c) It is a domain since it is open and connected. It is not
closed since {Im(z) 1} is not open and it is not bounded
since one can find a sequence zn satisfying Im(z) > 1 such that
limn |zn | = . For example, let zn = 2ni.

(d) It is not a domain since it is not open; choose a point z0


satisfying |z 4| = |z| and the disk |z z0 | < r is not contained
in the set for all r > 0. It is closed since {|z 4| < |z|} is open.
It is not bounded since one can find a sequence zn satisfying
|z 4| |z| such that limn |zn | = . For example, let
zn = ni.
(e) It is not a domain since it is not open; choose a point z0
satisfying Im(z) = 1 and the disk |z z0 | < r is not contained
in the set for all r > 0. It is closed since {Im(z) 6= 1} is open.
It is not bounded since one can find a sequence zn satisfying
Im(z) = 1 such that limn |zn | = . For example, let zn =
n + i.
(f) It is not a domain since it is not open; choose a point z0
satisfying arg(z) = 0 and the disk |z z0 | < r is not contained
in the set for all r > 0. It is not closed since 0 does not lie in
the set and the disk |z| < r meets the set for all r > 0. It is not
bounded since one can find a sequence zn satisfying arg(z) = 0
such that limn |zn | = . For example, let zn = n.
(7) Write the following functions f (z) in the forms f (z) = u(x, y)+
iv(x, y) under Cartesian coordinates and f (z) = u(r, )+iv(r, )
under polar coordinates:
(a) f (z) = z 2 + z + 1;
1+z
(b) f (z) =
.
1z
Solution. (a)
f (z) = (x + yi)2 + (x + yi) + 1 = (x2 y 2 + x + 1) + i(2xy + y)
= r2 e2i + rei + 1
= (r2 cos(2) + r cos + 1) + i(r2 sin(2) + r sin )
(b)
(1 + z)(1 z)
(1 z)(1 z)
1 + 2iy (x2 + y 2 )
=
(1 x)2 + y 2
1 (x2 + y 2 )
2y
=
+
i
(1 x)2 + y 2
(1 x)2 + y 2
1 r2
2r sin
=
+i
2
1 + r 2r cos
1 + r2 2r cos

f (z) =

(8) Sketch the region onto which the sector {r 1, 0 /4}


is mapped by the transformation (a) w = z 3 and (b) w = z 4 .
Solution. (a) The image is {|w| 1, 0 arg(w) 3/4}.
(b) The image is {|w| 1, 0 arg(w) }.

You might also like